Integral of an odd function over a symmetric interval

Click For Summary
The discussion centers on proving that the integral of an odd function over a symmetric interval, specifically ∫[-a, a] f(x) dx, equals zero. The proof begins by leveraging the property of odd functions, where f(-x) = -f(x), and involves splitting the integral into two parts. By substituting variables and applying the definition of odd functions, the terms cancel out, leading to the conclusion that the integral equals zero. Participants suggest alternative approaches, including defining a new function g(x) = f(-x) and considering Riemann sums to illustrate the cancellation of terms. The conversation highlights the challenge of demonstrating integrability for all odd functions over symmetric intervals.
sandra1
Messages
12
Reaction score
0

Homework Statement



f: [-a,a] >. R is Riemann integrable, prove that ∫[-a, a] ƒ (x) dx = 0

Homework Equations


The Attempt at a Solution



This only proof below I can think of is rather very calculus-ish.I wonder is there any other proof that is more Real Analysis level for this problem? Thanks alot.

since f is an odd function >> -f (x) = f (-x)

∫[-a, a] ƒ (x) dx = ∫[-a, 0] ƒ (x) dx + ∫[0, a] ƒ (x) dx
substitute t = -x with t is dummy variable.
∫[-a, a] ƒ (x) dx = ∫[a, 0] ƒ (-t) -dt + ∫[0, a] ƒ (x) dx (x = -a >> t = a, x = 0 >> t = 0)
= -∫[a, 0] ƒ (-t) dt + ∫[0,a] ƒ (x) dx = ∫[0, a] ƒ (-t) dt + ∫[0,a] ƒ (x) dx
t is dummy is we can rewrite as : = ∫[0,a] ƒ(-x) dx + ∫[0,a] ƒ(x) dx
= ∫[0, a] - ƒ (x) dt +∫[0, a] ƒ (x) dx
(by definition of odd function)
= -∫[0, a] ƒ (x) dt + ∫[0, a] ƒ (x) dx = 0
 
Physics news on Phys.org
hi sandra1! :wink:
sandra1 said:
This only proof below I can think of is rather very calculus-ish.

yes, particularly the "dummy variable"! :redface:

try defining a function g with g(x) = f(-x) :smile:
 
Alternatively, if you take a Riemann sum with evenly sized sub-intervals, the oddness of the function ends up canceling almost every term and it's not that hard to show that the integral must be zero (something to think about real quick: why doesn't this prove every odd function is integrable over symmetric intervals?)
 
Question: A clock's minute hand has length 4 and its hour hand has length 3. What is the distance between the tips at the moment when it is increasing most rapidly?(Putnam Exam Question) Answer: Making assumption that both the hands moves at constant angular velocities, the answer is ## \sqrt{7} .## But don't you think this assumption is somewhat doubtful and wrong?

Similar threads

  • · Replies 105 ·
4
Replies
105
Views
6K
Replies
6
Views
2K
Replies
8
Views
2K
  • · Replies 23 ·
Replies
23
Views
2K
Replies
9
Views
2K
Replies
2
Views
2K
  • · Replies 2 ·
Replies
2
Views
2K
  • · Replies 14 ·
Replies
14
Views
2K
  • · Replies 9 ·
Replies
9
Views
2K
Replies
4
Views
2K